Difference between revisions of "2024 AMC 12B Problems/Problem 17"

m (Solution 1)
m (Solution 1)
 
(2 intermediate revisions by the same user not shown)
Line 10: Line 10:
 
<math>-10 \leq a, b \leq 10</math>, each of <math>a,b</math> has <math>21</math> choices
 
<math>-10 \leq a, b \leq 10</math>, each of <math>a,b</math> has <math>21</math> choices
  
Applying Vieta,  
+
Applying Vieta's formulas,  
  
 
<math>x_1 \cdot x_2  \cdot x_3  = -6</math>
 
<math>x_1 \cdot x_2  \cdot x_3  = -6</math>
Line 30: Line 30:
 
(5)  <math> (x_1,x_2,x_3)  = (-1,-2,-3) , b = 11</math> invalid  
 
(5)  <math> (x_1,x_2,x_3)  = (-1,-2,-3) , b = 11</math> invalid  
  
the total event space is <math>21  \cdot (21- 1)</math> (choice of select a<math>\cdot</math>choice of selecting b given no-replacement)  
+
the total event space is <math>21  \cdot (21- 1)</math> (choice of select a times choice of selecting b given no-replacement)  
  
hence, our answer is <math>\frac{4}{21 \cdot 20}</math> <math>\boxed{\textbf{(C) }\frac{1}{105}}</math>
+
hence, our answer is <math>\frac{4}{21 \cdot 20} =  \boxed{\textbf{(C) }\frac{1}{105}}</math>
 
   
 
   
 
~[https://artofproblemsolving.com/wiki/index.php/User:Cyantist luckuso]
 
~[https://artofproblemsolving.com/wiki/index.php/User:Cyantist luckuso]

Latest revision as of 18:29, 16 November 2024

Problem 17

Integers $a$ and $b$ are randomly chosen without replacement from the set of integers with absolute value not exceeding $10$. What is the probability that the polynomial $x^3 + ax^2 + bx + 6$ has $3$ distinct integer roots?

$\textbf{(A) } \frac{1}{240} \qquad \textbf{(B) } \frac{1}{221} \qquad \textbf{(C) } \frac{1}{105} \qquad \textbf{(D) } \frac{1}{84} \qquad \textbf{(E) } \frac{1}{63}$.

Solution

Solution 1

$-10 \leq a, b \leq 10$, each of $a,b$ has $21$ choices

Applying Vieta's formulas,

$x_1 \cdot x_2  \cdot x_3  = -6$

$x_1 + x_2+ x_2 = -a$

$x_1 \cdot x_2 + x_1 \cdot x_3  + x_3 \cdot x_2  = b$

Cases:

(1) $(x_1,x_2,x_3)  = (-1,1,6) , b = -1, a=-6$ valid

(2) $(x_1,x_2,x_3)  = ( 1,2,-3) , b = -7, a=0$ valid

(3) $(x_1,x_2,x_3)  = (1,-2,3) , b = -7, a=2$ valid

(4) $(x_1,x_2,x_3)  = (-1,2,3) , b = 1, a=4$ valid

(5) $(x_1,x_2,x_3)  = (-1,-2,-3) , b = 11$ invalid

the total event space is $21  \cdot (21- 1)$ (choice of select a times choice of selecting b given no-replacement)

hence, our answer is $\frac{4}{21 \cdot 20} =   \boxed{\textbf{(C) }\frac{1}{105}}$

~luckuso

See also

2024 AMC 12B (ProblemsAnswer KeyResources)
Preceded by
Problem 16
Followed by
Problem 18
1 2 3 4 5 6 7 8 9 10 11 12 13 14 15 16 17 18 19 20 21 22 23 24 25
All AMC 12 Problems and Solutions

The problems on this page are copyrighted by the Mathematical Association of America's American Mathematics Competitions. AMC logo.png